=+ (c) Suppose Theorem 2.1 were weakened by strengthening its hypothesis to the assumption that is

Question:

=+

(c) Suppose Theorem 2.1 were weakened by strengthening its hypothesis to the assumption that ยง is a o-field. Why would this weakened result not suffice for the proof of Theorem 3.1?

Fantastic news! We've Found the answer you've been seeking!

Step by Step Answer:

Question Posted: